Prove every convergent sequence of real numbers is bounded &

In summary, the question asks to prove that every convergent sequence of real numbers is bounded, and that if the limit of the sequence is L, then the infimum of the sequence is less than or equal to L which is less than or equal to the supremum of the sequence. Part (a) shows that for a convergent sequence, L + 1 is not an upper bound and thus the sequence is bounded. Part (b) uses the fact that lim sup a[n] = lim inf a[n] = L for a convergent sequence, and shows that by assuming the infimum is greater than L, a contradiction is reached. Thus, the desired result is proved.
  • #1
STEMucator
Homework Helper
2,076
140

Homework Statement



The question : http://gyazo.com/7eb4b86c61150e4af092b9f8afeaf169

Homework Equations



Sup/Inf axioms
Methods of constructing sequences
##ε-N##
##lim(a_n) ≤ sup_n a_n## from question 5 right before it.

I'll split the question into two parts.

The Attempt at a Solution



(a) We must prove every convergent sequence of real numbers is bounded. Suppose ##a_n## is a convergent sequence, that is ##lim(a_n) = L## where ##L \in ℝ##.

So for ## ε = 1, \exists N \space | \space n ≥ N \Rightarrow |a_n - L| < 1##

Thus ##-1 + L < a_n < 1 + L## so that ##a_n## is bounded below by -1 + L and above by 1 + L.

(b) Suppose ##lim(a_n) = L##, we must prove that ##inf_n a_n ≤ L ≤ sup_n a_n##

From question 5 prior, we already know that ##L ≤ sup_n a_n##

So we want to show ##inf_n a_n ≤ L##.

Assume the converse is true, that is suppose that ##inf_n a_n > L##.

Since ##inf_n a_n ≤ sup_n a_n##, we have that ##sup_n a_n ≥ inf_n a_n > L##, but this implies that ##sup_n a_n > L## which is a contradiction.
 
Physics news on Phys.org
  • #2
Part a)
This does not in general work. All you know is that for [itex]n\geq N[/itex], this bound is valid. What about for [itex]n<N[/itex]?
 
  • #3
notice that a sequence {a[n]} is convergent if lim sup a[n] = lim inf a[n] = L. the idea is that as 'n' goes to infinity, the largest or smallest value in the 'tail' of the sequence approaches L monotonically [why?]. so if "inf a[n] > L" for some value of 'n', if must be true for all larger values of 'n'.
 
  • #4
And for (b), what is the contradiction? There is no problem with ##\sup a_n > L##. Also, to make your posts more readable, use a backslash in front of keywords like inf, sup, lim.
 
  • #5
christoff said:
Part a)
This does not in general work. All you know is that for [itex]n\geq N[/itex], this bound is valid. What about for [itex]n<N[/itex]?

I'll address this first.

After some thought I believe i see what you're intending. I don't need to consider n<N. I just need to choose a different N, say ##N'##

(a) We must prove every convergent sequence of real numbers is bounded. Suppose ##a_n## is a convergent sequence, that is ##\lim(a_n) = L## where ##L \in ℝ##.

So for ## ε = 1, \exists N \space | \space n ≥ N \Rightarrow |a_n - L| < 1##

Thus ##-1 + L < a_n < 1 + L## so that ##a_n## is bounded below by -1 + L and above by 1 + L. This covers all cases of ##n ≥ N##.

Now, for some ##n < N## we can see that ##a_n > L + 1## so that ##L + 1## is not an upper bound so we define :

##M = max\{ max\{a_n \space | \space n < N\}, L + 1 \}## so we could bound ##a_n## as desired.

(b) Suppose ##\lim(a_n) = L##, we must prove that ##\inf_n a_n ≤ L ≤ \sup_n a_n##

From question 5 prior, we already know that ##L ≤ \sup_n a_n##

So we want to show ##\inf_n a_n ≤ L##.

Assume the converse is true, that is suppose that ##\inf_n a_n > L##.

We know ##\forall ε > 0, \exists N \space | \space n ≥ N \Rightarrow |a_n - L| < ε##

So for ##ε = \inf_n a_n - L## we have ## |L - a_n| < \inf_n a_n - L##

So we have ##-\inf_n a_n + L < a_n - L < \inf_n a_n - L## which yields ##a_n < \inf_n a_n## which is a contradiction.

##∴ \inf_n a_n ≤ \lim(a_n) ≤ \sup_n a_n## as desired.

OUT OF QUESTION : Is ##\sup_n a_n = limsup(a_n)##? Ditto for liminf?
 
Last edited:
  • #6
Zondrina said:
I'll address this first.

After some thought I believe i see what you're intending. I don't need to consider n<N. I just need to choose a different N, say ##N'##

(a) Suppose ##a_n## is a convergent sequence so that ##lim(a_n) = L##.

So we know ##\forall ε > 0, \exists N \space | \space n > N \Rightarrow |a_n - L| < ε##.

We want to show ##a_n## is bounded, that is we must show that ##|a_n| ≤ M## for some ##M \in ℝ##

So let's choose an ε. For ##ε = 1, \exists N' \space | \space n > N' \Rightarrow |a_n - L| < 1##

Now we can use the fact ##|x| - |y| ≤ |x - y|## so we get the inequality ##|a_n| - |L| ≤ |a_n - L| < 1##.

So we can take the portion of the inequality ##|a_n| - |L| < 1## and conclude that ##|a_n| < 1 + |L|##.

So choosing ##M = max\{a_n, 1 + |L| \space | \space n \in \mathbb{N}\}##, ( We have to consider all the terms in the sequence as well ) we can bound ##a_n## as desired.

Well, you took that hint completely the wrong way. I don't see what N' is buying you. You've already shown ##a_n## is bounded for n>N. All you have to worry about is n<=N. That isn't so hard to deal with is it? There are only a finite number of terms.
 
  • #7
Dick said:
Well, you took that hint completely the wrong way. I don't see what N' is buying you. You've already shown ##a_n## is bounded for nN. All you have to worry about is n<N. That isn't so hard to deal with is it? There are only a finite number of terms.

Hmm you're right, now that I look back I see I didn't really need N', but why is my ##M## not good?

I'm not quite sure why I have to consider everything below a certain number when I'm considering all terms of the sequence as well as ##ε + |L|## for some positive ##ε##.

Also I added another try at part (b).

EDIT : I see I don't need my ##N_2## either now.

EDIT 2 : Wow facepalm, i just realized what n<N meant. I fixed my M accordingly.

EDIT 3 : Fixed all the errors now hopefully.
 
Last edited:
  • #8
Zondrina said:
Hmm you're right, now that I look back I see I didn't really need N', but why is my ##M## not good?

I'm not quite sure why I have to consider everything below a certain number when I'm considering all terms of the sequence as well as ##ε + |L|## for some positive ##ε##.

Also I added another try at part (b).

EDIT : I see I don't need my ##N_2## either now.

EDIT 2 : Wow facepalm, i just realized what n<N meant. I fixed my M accordingly.

EDIT 3 : Fixed all the errors now hopefully.

It's really hard to tell what you changed. It's still all a mess. Look, your initial try at part a) was almost correct. Just say what you are going to do with n<=N. You can't take the max over all ##a_n## until you shown it has a max. Otherwise you could have done that to begin with.
 
Last edited:
  • #9
Dick said:
It's really hard to tell what you changed. It's still all a mess. Look, your initial try at part a) was almost correct. Just say what you are going to do with n<=N.

I dropped the unneeded N's and cleaned it up a bit.

Also my initial try? As in the one in my first post? I don't see how my most recent attempt is wrong considering I took ##M = \{|a_n|, 1 + |L| ... ## ( I wrapped ##a_n## in an abs ). This means that I consider all values of ##n \in \mathbb{N}##, not just values ##n ≥ N## ( I understand your issue is that I need to show ##a_n## is bounded for all n ). So I'm not convinced this is incorrect.

Considering my initial try was correct though... let me try to reason out what I would do for n < N ( chris also mentioned this in an earlier post ).

Hmm for some ##n < N## we can see that ##a_n > L + 1## so that ##L + 1## is not an upper bound so I would define :

##M = max\{ max\{a_n \space | \space n < N\}, L + 1 \}## so we could bound ##a_n## as desired.

I still don't see what's wrong with what I mentioned prior though.
 
  • #10
Zondrina said:
I dropped the unneeded N's and cleaned it up a bit.

Also my initial try? As in the one in my first post? I don't see how my most recent attempt is wrong considering I took ##M = \{|a_n|, 1 + |L| ... ## ( I wrapped ##a_n## in an abs ). This means that I consider all values of ##n \in \mathbb{N}##, not just values ##n ≥ N## ( I understand your issue is that I need to show ##a_n## is bounded for all n ). So I'm not convinced this is incorrect.

Considering my initial try was correct though... let me try to reason out what I would do for n < N ( chris also mentioned this in an earlier post ).

Hmm for some ##n < N## we can see that ##a_n > L + 1## so that ##L + 1## is not an upper bound so I would define :

##M = max\{ max\{a_n \space | \space n < N\}, L + 1 \}## so we could bound ##a_n## as desired.

I still don't see what's wrong with what I mentioned prior though.

max of ##a_n## for n<N is well defined regardless of whether the series even converges or not. You are taking the max of a FINITE number of terms. That's why it's well defined. max ##a_n## over all n is not. Until you say why it is. Your latest attempt makes that clear.
 
  • #11
Dick said:
max of ##a_n## for n<N is well defined regardless of whether the series even converges or not. You are taking the max of a FINITE number of terms. That's why it's well defined. max ##a_n## over all n is not. Until you say why it is. Your latest attempt makes that clear.

Ahh yes I understand your concern now. So I can't just use the max over what may potentially be an infinite number of terms as... that's simply broken.

EDIT : Cleaned up the proof in post #5, should be good now.
 
Last edited:

1. What does it mean for a sequence of real numbers to be convergent?

A sequence of real numbers is said to be convergent if its terms approach a specific limit as the sequence progresses, meaning that the terms become closer and closer to the limit value as the sequence continues.

2. How can we prove that every convergent sequence of real numbers is bounded?

We can prove that every convergent sequence of real numbers is bounded by using the definition of convergence. Since the terms of a convergent sequence get closer and closer to the limit, we can choose a value that is greater than the distance between any term and the limit to serve as an upper bound for the sequence.

3. Is it possible for a convergent sequence of real numbers to be unbounded?

No, it is not possible for a convergent sequence of real numbers to be unbounded. This is because the definition of convergence requires the terms to approach a limit, meaning that the sequence must have a finite bound.

4. Can a sequence of real numbers be bounded but not convergent?

Yes, a sequence of real numbers can be bounded but not convergent. This occurs when the terms of the sequence are limited to a certain range, but do not approach a specific limit as the sequence progresses.

5. How does the concept of boundedness relate to the convergence of a sequence of real numbers?

The concept of boundedness is closely related to convergence in that a convergent sequence must be bounded. This is because the terms of a convergent sequence get closer and closer to a specific limit, meaning that they are limited to a certain range and cannot have infinite values.

Similar threads

  • Calculus and Beyond Homework Help
Replies
2
Views
189
  • Calculus and Beyond Homework Help
Replies
1
Views
259
  • Calculus and Beyond Homework Help
Replies
4
Views
884
  • Calculus and Beyond Homework Help
Replies
2
Views
711
  • Calculus and Beyond Homework Help
Replies
34
Views
2K
  • Calculus and Beyond Homework Help
Replies
3
Views
843
  • Calculus and Beyond Homework Help
Replies
3
Views
1K
  • Calculus and Beyond Homework Help
Replies
8
Views
2K
  • Calculus and Beyond Homework Help
Replies
5
Views
1K
  • Calculus and Beyond Homework Help
Replies
6
Views
2K
Back
Top